Gradient of $||Ax - y||^2$ with respect to $A$












1














How do I proceed to find $nabla_A||Ax - y||^2$ where $A in mathbb{R}^{ntimes n}$ and $x,y in mathbb{R}^n$ and the norm is the Euclidean norm.



Attempt so far



$$||Ax - y||^2 = (Ax-y)^T(Ax-y) = x^TA^TAx - 2x^TAy + y^Ty $$



$$ nabla_A(x^TAy) = xy^T$$



Where I am stuck



I don't know how to tackle the $x^TA^TAx$ term since if I try to apply chain rule, I will have to differentiate a matrix with respect to a matrix.










share|cite|improve this question





























    1














    How do I proceed to find $nabla_A||Ax - y||^2$ where $A in mathbb{R}^{ntimes n}$ and $x,y in mathbb{R}^n$ and the norm is the Euclidean norm.



    Attempt so far



    $$||Ax - y||^2 = (Ax-y)^T(Ax-y) = x^TA^TAx - 2x^TAy + y^Ty $$



    $$ nabla_A(x^TAy) = xy^T$$



    Where I am stuck



    I don't know how to tackle the $x^TA^TAx$ term since if I try to apply chain rule, I will have to differentiate a matrix with respect to a matrix.










    share|cite|improve this question



























      1












      1








      1







      How do I proceed to find $nabla_A||Ax - y||^2$ where $A in mathbb{R}^{ntimes n}$ and $x,y in mathbb{R}^n$ and the norm is the Euclidean norm.



      Attempt so far



      $$||Ax - y||^2 = (Ax-y)^T(Ax-y) = x^TA^TAx - 2x^TAy + y^Ty $$



      $$ nabla_A(x^TAy) = xy^T$$



      Where I am stuck



      I don't know how to tackle the $x^TA^TAx$ term since if I try to apply chain rule, I will have to differentiate a matrix with respect to a matrix.










      share|cite|improve this question















      How do I proceed to find $nabla_A||Ax - y||^2$ where $A in mathbb{R}^{ntimes n}$ and $x,y in mathbb{R}^n$ and the norm is the Euclidean norm.



      Attempt so far



      $$||Ax - y||^2 = (Ax-y)^T(Ax-y) = x^TA^TAx - 2x^TAy + y^Ty $$



      $$ nabla_A(x^TAy) = xy^T$$



      Where I am stuck



      I don't know how to tackle the $x^TA^TAx$ term since if I try to apply chain rule, I will have to differentiate a matrix with respect to a matrix.







      vector-analysis matrix-calculus






      share|cite|improve this question















      share|cite|improve this question













      share|cite|improve this question




      share|cite|improve this question








      edited Nov 22 at 9:31









      user550103

      6711315




      6711315










      asked Nov 22 at 6:38









      Anant Joshi

      451211




      451211






















          2 Answers
          2






          active

          oldest

          votes


















          1














          Before we start deriving the gradient, some facts and notations for brevity:




          • Trace and Frobenius product relation $$leftlangle A, B Crightrangle={rm tr}(A^TBC) := A : B C$$

          • Cyclic properties of Trace/Frobenius product
            begin{align}
            A : B C
            &= BC : A \
            &= A C^T : B \
            &= {text{etc.}} cr
            end{align}



          Let $f := left|Ax-y right|^2 = Ax-y:Ax-y$.



          Now, we can obtain the differential first, and then the gradient.
          begin{align}
          df
          &= dleft( Ax-y:Ax-y right) \
          &= left(dA x : Ax-yright) + left(Ax-y : dA xright) \
          &= 2 left(Ax - yright) : dA x \
          &= 2left( Ax-yright)x^T : dA\
          end{align}



          Thus, the gradient is
          begin{align}
          frac{partial}{partial A} left( left|Ax-y right|^2 right)= 2left( Ax-yright)x^T.
          end{align}






          share|cite|improve this answer























          • What is the difference between differential and gradient?
            – Anant Joshi
            Nov 22 at 9:19










          • see this math.stackexchange.com/questions/289923/…
            – user550103
            Nov 22 at 9:20



















          0














          For matrices, the most easy way is often to get back to definition of differentiability, i.e. :



          $$||(A+H)x - y||^2-||Ax - y||^2=L_A(H) + o(|H|)$$



          With $L_A$ a linear map.



          We begin with :
          $$||(A+H)x - y||^2=langle Ax+Hx-y , Ax+Hx-y rangle.$$



          Then we have :



          $$||(A+H)x - y||^2= langle Ax-y,Ax-yrangle + 2langle Hx,Ax -yrangle + langle Hx,Hxrangle.$$



          To conclude : $$||(A+H)x - y||^2-||Ax - y||^2=2langle Hx,Ax -yrangle + o (|H|)$$



          Thus :



          $$left(nabla_A||Ax - y||^2right)_{i,j}=2langle E_{i,j}x,Ax -yrangle$$



          With $E_{i,j}$ the matrix with a $1$ at row $i$ and column $j$ and $0$ otherwise.






          share|cite|improve this answer























          • Why is the final answer a number and not a matrix?
            – Anant Joshi
            Nov 22 at 9:13










          • Because I misunderstood the notation and went a bit fast at the end. Is it better now ?
            – nicomezi
            Nov 22 at 9:21











          Your Answer





          StackExchange.ifUsing("editor", function () {
          return StackExchange.using("mathjaxEditing", function () {
          StackExchange.MarkdownEditor.creationCallbacks.add(function (editor, postfix) {
          StackExchange.mathjaxEditing.prepareWmdForMathJax(editor, postfix, [["$", "$"], ["\\(","\\)"]]);
          });
          });
          }, "mathjax-editing");

          StackExchange.ready(function() {
          var channelOptions = {
          tags: "".split(" "),
          id: "69"
          };
          initTagRenderer("".split(" "), "".split(" "), channelOptions);

          StackExchange.using("externalEditor", function() {
          // Have to fire editor after snippets, if snippets enabled
          if (StackExchange.settings.snippets.snippetsEnabled) {
          StackExchange.using("snippets", function() {
          createEditor();
          });
          }
          else {
          createEditor();
          }
          });

          function createEditor() {
          StackExchange.prepareEditor({
          heartbeatType: 'answer',
          autoActivateHeartbeat: false,
          convertImagesToLinks: true,
          noModals: true,
          showLowRepImageUploadWarning: true,
          reputationToPostImages: 10,
          bindNavPrevention: true,
          postfix: "",
          imageUploader: {
          brandingHtml: "Powered by u003ca class="icon-imgur-white" href="https://imgur.com/"u003eu003c/au003e",
          contentPolicyHtml: "User contributions licensed under u003ca href="https://creativecommons.org/licenses/by-sa/3.0/"u003ecc by-sa 3.0 with attribution requiredu003c/au003e u003ca href="https://stackoverflow.com/legal/content-policy"u003e(content policy)u003c/au003e",
          allowUrls: true
          },
          noCode: true, onDemand: true,
          discardSelector: ".discard-answer"
          ,immediatelyShowMarkdownHelp:true
          });


          }
          });














          draft saved

          draft discarded


















          StackExchange.ready(
          function () {
          StackExchange.openid.initPostLogin('.new-post-login', 'https%3a%2f%2fmath.stackexchange.com%2fquestions%2f3008814%2fgradient-of-ax-y2-with-respect-to-a%23new-answer', 'question_page');
          }
          );

          Post as a guest















          Required, but never shown

























          2 Answers
          2






          active

          oldest

          votes








          2 Answers
          2






          active

          oldest

          votes









          active

          oldest

          votes






          active

          oldest

          votes









          1














          Before we start deriving the gradient, some facts and notations for brevity:




          • Trace and Frobenius product relation $$leftlangle A, B Crightrangle={rm tr}(A^TBC) := A : B C$$

          • Cyclic properties of Trace/Frobenius product
            begin{align}
            A : B C
            &= BC : A \
            &= A C^T : B \
            &= {text{etc.}} cr
            end{align}



          Let $f := left|Ax-y right|^2 = Ax-y:Ax-y$.



          Now, we can obtain the differential first, and then the gradient.
          begin{align}
          df
          &= dleft( Ax-y:Ax-y right) \
          &= left(dA x : Ax-yright) + left(Ax-y : dA xright) \
          &= 2 left(Ax - yright) : dA x \
          &= 2left( Ax-yright)x^T : dA\
          end{align}



          Thus, the gradient is
          begin{align}
          frac{partial}{partial A} left( left|Ax-y right|^2 right)= 2left( Ax-yright)x^T.
          end{align}






          share|cite|improve this answer























          • What is the difference between differential and gradient?
            – Anant Joshi
            Nov 22 at 9:19










          • see this math.stackexchange.com/questions/289923/…
            – user550103
            Nov 22 at 9:20
















          1














          Before we start deriving the gradient, some facts and notations for brevity:




          • Trace and Frobenius product relation $$leftlangle A, B Crightrangle={rm tr}(A^TBC) := A : B C$$

          • Cyclic properties of Trace/Frobenius product
            begin{align}
            A : B C
            &= BC : A \
            &= A C^T : B \
            &= {text{etc.}} cr
            end{align}



          Let $f := left|Ax-y right|^2 = Ax-y:Ax-y$.



          Now, we can obtain the differential first, and then the gradient.
          begin{align}
          df
          &= dleft( Ax-y:Ax-y right) \
          &= left(dA x : Ax-yright) + left(Ax-y : dA xright) \
          &= 2 left(Ax - yright) : dA x \
          &= 2left( Ax-yright)x^T : dA\
          end{align}



          Thus, the gradient is
          begin{align}
          frac{partial}{partial A} left( left|Ax-y right|^2 right)= 2left( Ax-yright)x^T.
          end{align}






          share|cite|improve this answer























          • What is the difference between differential and gradient?
            – Anant Joshi
            Nov 22 at 9:19










          • see this math.stackexchange.com/questions/289923/…
            – user550103
            Nov 22 at 9:20














          1












          1








          1






          Before we start deriving the gradient, some facts and notations for brevity:




          • Trace and Frobenius product relation $$leftlangle A, B Crightrangle={rm tr}(A^TBC) := A : B C$$

          • Cyclic properties of Trace/Frobenius product
            begin{align}
            A : B C
            &= BC : A \
            &= A C^T : B \
            &= {text{etc.}} cr
            end{align}



          Let $f := left|Ax-y right|^2 = Ax-y:Ax-y$.



          Now, we can obtain the differential first, and then the gradient.
          begin{align}
          df
          &= dleft( Ax-y:Ax-y right) \
          &= left(dA x : Ax-yright) + left(Ax-y : dA xright) \
          &= 2 left(Ax - yright) : dA x \
          &= 2left( Ax-yright)x^T : dA\
          end{align}



          Thus, the gradient is
          begin{align}
          frac{partial}{partial A} left( left|Ax-y right|^2 right)= 2left( Ax-yright)x^T.
          end{align}






          share|cite|improve this answer














          Before we start deriving the gradient, some facts and notations for brevity:




          • Trace and Frobenius product relation $$leftlangle A, B Crightrangle={rm tr}(A^TBC) := A : B C$$

          • Cyclic properties of Trace/Frobenius product
            begin{align}
            A : B C
            &= BC : A \
            &= A C^T : B \
            &= {text{etc.}} cr
            end{align}



          Let $f := left|Ax-y right|^2 = Ax-y:Ax-y$.



          Now, we can obtain the differential first, and then the gradient.
          begin{align}
          df
          &= dleft( Ax-y:Ax-y right) \
          &= left(dA x : Ax-yright) + left(Ax-y : dA xright) \
          &= 2 left(Ax - yright) : dA x \
          &= 2left( Ax-yright)x^T : dA\
          end{align}



          Thus, the gradient is
          begin{align}
          frac{partial}{partial A} left( left|Ax-y right|^2 right)= 2left( Ax-yright)x^T.
          end{align}







          share|cite|improve this answer














          share|cite|improve this answer



          share|cite|improve this answer








          edited Nov 22 at 9:17

























          answered Nov 22 at 9:08









          user550103

          6711315




          6711315












          • What is the difference between differential and gradient?
            – Anant Joshi
            Nov 22 at 9:19










          • see this math.stackexchange.com/questions/289923/…
            – user550103
            Nov 22 at 9:20


















          • What is the difference between differential and gradient?
            – Anant Joshi
            Nov 22 at 9:19










          • see this math.stackexchange.com/questions/289923/…
            – user550103
            Nov 22 at 9:20
















          What is the difference between differential and gradient?
          – Anant Joshi
          Nov 22 at 9:19




          What is the difference between differential and gradient?
          – Anant Joshi
          Nov 22 at 9:19












          see this math.stackexchange.com/questions/289923/…
          – user550103
          Nov 22 at 9:20




          see this math.stackexchange.com/questions/289923/…
          – user550103
          Nov 22 at 9:20











          0














          For matrices, the most easy way is often to get back to definition of differentiability, i.e. :



          $$||(A+H)x - y||^2-||Ax - y||^2=L_A(H) + o(|H|)$$



          With $L_A$ a linear map.



          We begin with :
          $$||(A+H)x - y||^2=langle Ax+Hx-y , Ax+Hx-y rangle.$$



          Then we have :



          $$||(A+H)x - y||^2= langle Ax-y,Ax-yrangle + 2langle Hx,Ax -yrangle + langle Hx,Hxrangle.$$



          To conclude : $$||(A+H)x - y||^2-||Ax - y||^2=2langle Hx,Ax -yrangle + o (|H|)$$



          Thus :



          $$left(nabla_A||Ax - y||^2right)_{i,j}=2langle E_{i,j}x,Ax -yrangle$$



          With $E_{i,j}$ the matrix with a $1$ at row $i$ and column $j$ and $0$ otherwise.






          share|cite|improve this answer























          • Why is the final answer a number and not a matrix?
            – Anant Joshi
            Nov 22 at 9:13










          • Because I misunderstood the notation and went a bit fast at the end. Is it better now ?
            – nicomezi
            Nov 22 at 9:21
















          0














          For matrices, the most easy way is often to get back to definition of differentiability, i.e. :



          $$||(A+H)x - y||^2-||Ax - y||^2=L_A(H) + o(|H|)$$



          With $L_A$ a linear map.



          We begin with :
          $$||(A+H)x - y||^2=langle Ax+Hx-y , Ax+Hx-y rangle.$$



          Then we have :



          $$||(A+H)x - y||^2= langle Ax-y,Ax-yrangle + 2langle Hx,Ax -yrangle + langle Hx,Hxrangle.$$



          To conclude : $$||(A+H)x - y||^2-||Ax - y||^2=2langle Hx,Ax -yrangle + o (|H|)$$



          Thus :



          $$left(nabla_A||Ax - y||^2right)_{i,j}=2langle E_{i,j}x,Ax -yrangle$$



          With $E_{i,j}$ the matrix with a $1$ at row $i$ and column $j$ and $0$ otherwise.






          share|cite|improve this answer























          • Why is the final answer a number and not a matrix?
            – Anant Joshi
            Nov 22 at 9:13










          • Because I misunderstood the notation and went a bit fast at the end. Is it better now ?
            – nicomezi
            Nov 22 at 9:21














          0












          0








          0






          For matrices, the most easy way is often to get back to definition of differentiability, i.e. :



          $$||(A+H)x - y||^2-||Ax - y||^2=L_A(H) + o(|H|)$$



          With $L_A$ a linear map.



          We begin with :
          $$||(A+H)x - y||^2=langle Ax+Hx-y , Ax+Hx-y rangle.$$



          Then we have :



          $$||(A+H)x - y||^2= langle Ax-y,Ax-yrangle + 2langle Hx,Ax -yrangle + langle Hx,Hxrangle.$$



          To conclude : $$||(A+H)x - y||^2-||Ax - y||^2=2langle Hx,Ax -yrangle + o (|H|)$$



          Thus :



          $$left(nabla_A||Ax - y||^2right)_{i,j}=2langle E_{i,j}x,Ax -yrangle$$



          With $E_{i,j}$ the matrix with a $1$ at row $i$ and column $j$ and $0$ otherwise.






          share|cite|improve this answer














          For matrices, the most easy way is often to get back to definition of differentiability, i.e. :



          $$||(A+H)x - y||^2-||Ax - y||^2=L_A(H) + o(|H|)$$



          With $L_A$ a linear map.



          We begin with :
          $$||(A+H)x - y||^2=langle Ax+Hx-y , Ax+Hx-y rangle.$$



          Then we have :



          $$||(A+H)x - y||^2= langle Ax-y,Ax-yrangle + 2langle Hx,Ax -yrangle + langle Hx,Hxrangle.$$



          To conclude : $$||(A+H)x - y||^2-||Ax - y||^2=2langle Hx,Ax -yrangle + o (|H|)$$



          Thus :



          $$left(nabla_A||Ax - y||^2right)_{i,j}=2langle E_{i,j}x,Ax -yrangle$$



          With $E_{i,j}$ the matrix with a $1$ at row $i$ and column $j$ and $0$ otherwise.







          share|cite|improve this answer














          share|cite|improve this answer



          share|cite|improve this answer








          edited Nov 22 at 9:20

























          answered Nov 22 at 9:11









          nicomezi

          4,0741820




          4,0741820












          • Why is the final answer a number and not a matrix?
            – Anant Joshi
            Nov 22 at 9:13










          • Because I misunderstood the notation and went a bit fast at the end. Is it better now ?
            – nicomezi
            Nov 22 at 9:21


















          • Why is the final answer a number and not a matrix?
            – Anant Joshi
            Nov 22 at 9:13










          • Because I misunderstood the notation and went a bit fast at the end. Is it better now ?
            – nicomezi
            Nov 22 at 9:21
















          Why is the final answer a number and not a matrix?
          – Anant Joshi
          Nov 22 at 9:13




          Why is the final answer a number and not a matrix?
          – Anant Joshi
          Nov 22 at 9:13












          Because I misunderstood the notation and went a bit fast at the end. Is it better now ?
          – nicomezi
          Nov 22 at 9:21




          Because I misunderstood the notation and went a bit fast at the end. Is it better now ?
          – nicomezi
          Nov 22 at 9:21


















          draft saved

          draft discarded




















































          Thanks for contributing an answer to Mathematics Stack Exchange!


          • Please be sure to answer the question. Provide details and share your research!

          But avoid



          • Asking for help, clarification, or responding to other answers.

          • Making statements based on opinion; back them up with references or personal experience.


          Use MathJax to format equations. MathJax reference.


          To learn more, see our tips on writing great answers.





          Some of your past answers have not been well-received, and you're in danger of being blocked from answering.


          Please pay close attention to the following guidance:


          • Please be sure to answer the question. Provide details and share your research!

          But avoid



          • Asking for help, clarification, or responding to other answers.

          • Making statements based on opinion; back them up with references or personal experience.


          To learn more, see our tips on writing great answers.




          draft saved


          draft discarded














          StackExchange.ready(
          function () {
          StackExchange.openid.initPostLogin('.new-post-login', 'https%3a%2f%2fmath.stackexchange.com%2fquestions%2f3008814%2fgradient-of-ax-y2-with-respect-to-a%23new-answer', 'question_page');
          }
          );

          Post as a guest















          Required, but never shown





















































          Required, but never shown














          Required, but never shown












          Required, but never shown







          Required, but never shown

































          Required, but never shown














          Required, but never shown












          Required, but never shown







          Required, but never shown







          Popular posts from this blog

          Plaza Victoria

          In PowerPoint, is there a keyboard shortcut for bulleted / numbered list?

          How to put 3 figures in Latex with 2 figures side by side and 1 below these side by side images but in...